CFP - Retirement Planning & Employee Benefits

Pataasin ang iyong marka sa homework at exams ngayon gamit ang Quizwiz!

Which of the following statements are true in regards to Section 457 plans? Eligible plan sponsors include non-profit organizations, churches, and governmental entities. In-service distributions after age 59 1/2 are allowed in a 457 plan. Salary deferrals are subject to Social Security, Medicare, and Federal unemployment tax in the year of the deferral. Assets of the plans for non-government entities are subject to the claims of the sponsor's general creditors.

Churches are not qualifying sponsors of 457 plans. In-service distributions are not allowed until age 70 1/2.

Jeb, age 54, works for Gamma Corporation and Epsilon Corporation. Gamma and Epsilon are both part of the same parent-subsidiary control group. Gamma and Epsilon both sponsor a 25% money purchase plan. If Jeb earns $200,000 at Gamma and $30,000 at Epsilon what is the maximum employer contribution that can be made to both plans? $56,000 $57,500 $61,500 $81,000

$56k. Notice that this is an employer contribution plan (DC PSP) don't get distracted by the other stuff. Max contribution for PSP is total of $56k.

Qualified Plans have what?

1) Tax deferral 2)payroll tax saving 3)federally provided creditor asset protection

What is a highly compensated employee?

1) more than 5% owner at any time during plan year or preceding plan year 2) employee compensation in excess of $125k for prior plan year

Which of the following vesting schedules may a non-top-heavy profit sharing plan use? 2 to 6 year graduated. 3-year cliff. 1 to 4 year graduated. 3 to 7 year cliff.

As a result of the PPA 2006, a profit sharing plan must vest at least as rapidly as a 3-year cliff or 2 to 6 year graduated schedule without regard to the plan's top-heavy status. The profit sharing plan can follow any vesting schedule that provides a more generous vesting schedule.

Generally, younger entrants are favored in which of the following plans? Defined benefit pension plans. Cash balance pension plans. Target benefit pension plans. Money purchase pension plans.

Cash Balance and Money Purchase Pension Plans favor younger entrants. Defined Benefit and Target Benefit Pension Plans favor older age entrants with less time to accumulate, and therefore, require higher funding levels.

Which of the following legal requirements apply to Employee Stock Ownership Plans (ESOPs)? ESOPs must permit participants, who are aged 55 or older and who have at least 10 years of service, the opportunity to diversify their accounts. ESOPs can be integrated with Social Security. An employer's deduction for ESOP contributions and amounts made to repay interest on an ESOP's debt cannot exceed 25% of the participant's payroll. The mandatory 20% income tax withholding requirement does not apply to distributions of employer stock from an ESOP.

Deductions for interest payments are not limited for ESOP plans. Deductions for repayment of principal is limited to 25% of covered compensation. must let someone age 55 older and 10 years of service diversify

SEP-IRA plans are unique from defined contribution plans in which of the following areas: Length of permissible exclusion from coverage based upon service. Establishment date of the plan. Income requirements for participation. Can be paired with another plan.

Employees can be excluded up to 3 years or age 21, whichever is longer. Plans can be established and funded up to the date of filing the entity tax return, including extensions. Employee needs to earn only $600 to be included in the plan.

Your client, Sue, age 35, is covered by a pension plan at work, but her spouse, age 37, is not covered by a pension plan. Her salary is $45,000 and his salary is $50,000. How much will go into his account if he contributes the maximum amount to a maximum funded, matching SIMPLE IRA?

He can contribute $13,000 (2019) and his employer will match $1 for $1 up to 3% of salary ($50,000 × .03 = $1,500). Therefore, maximum contribution is $13,000 + $1,500 = $14,500.

Bobby's Bar-b-que wants to establish a social security integrated plan using the offset method. Which of the following plans should he establish? SIMPLE ESOP Money Purchase Pension Plan Defined Benefit Pension Plan

He should establish the Defined Benefit Pension Plan. Only defined benefit plans can use the offset method. The Money Purchase Pension plan is a Defined Contribution Plan and must use the excess method. Simple's and ESOPs cannot be integrated.

Profit Sharing Plans Vesting

Must use a graded 2-6 or 3 year cliff at the minimum.

Timothy is covered under his employer's Defined Benefit Pension Plan. He earns $500,000 per year. The Defined Benefit Plan uses a funding formula of Years of Service × Average of Three Highest Years of Compensation × 2%. He has been with the employer for 25 years. What is the maximum contribution that can be made to the plan on his behalf? $132,500 $225,000 $280,000 It is indeterminable from the information given.

Read the question carefully. The question asks "what is the maximum contribution that can be made." Remember that for a defined pension plan the contribution must be whatever the actuary determines needs to be made to the plan.

Which of the following is a correct statement about the income tax implications of employer premium payments for group health insurance? An S Corporation can only deduct 70% of the premiums for all employees. In a sole proprietorship, the premiums for both the owner and the non-owner are fully deductible. If stockholder/employees of a closely held C corporation are covered as employees, the premiums are fully deductible. Premium costs paid by a partnership are passed through to the partner, who can deduct 70% of the costs on their individual tax returns.

S Corporations and proprietorships cannot deduct any premiums for group health insurance for owners. Non-owner employee health premiums are fully deductible to both entities. Answer "D" is incorrect because partners are able to deduct 100% of the health insurance premium on their individual tax returns.

Complex Corporation is ready to adopt a profit sharing plan for eligible employees. Which of the following groups would have to be considered in meeting the statutory coverage and participation tests? Employees of Simple Corporation, in which Complex owns 85% of the stock. Employees of Universal Corporation, in which Complex owns 55% of the stock. Rank and file workers at Complex who are union members with a contract that provides retirement benefits as a result of good-faith collective bargaining. Employees who are leased and covered by the leasing corporation's profit sharing plan.

Simple must be considered because Complex owns more than 80% and the leased employees must be considered because their leasing company's is not a pension plan. Universal would not be considered a subsidiary because it is only 55% not more than 80%. The union employees are excluded from testing by the IRC.

Which of the following statements is/are characteristic(s) of Tax-Sheltered annuities (TSAs)? Salary reduction contributions are NOT reported as W-2 income and are subject to Social Security tax. The maximum salary deferral limit is $19,000 for a newly hired employee in 2019, under age 50. Employer contributions are deductible by the employer. Loans and "catch-up" contributions may be permitted

TSA salary deductions are subject to Social Security (payroll) taxes; contributions by the employer to the TSA are tax deductible by the employer but these contributions are not subject to payroll taxes, much like 401K contributions. Remember, TSAs are also known as a 403(b) retirement plan. All are true

Robert Sullivan, age 56, works for Dynex Corporation, and earns $285,000. Dynex Corp. provides a non-elective 2% contribution to its SIMPLE IRA plan. Which one of the following is the maximum amount that could go into Robert's account this year?

The compensation limit of $280,000 applies to SIMPLE IRAs when non-elective contributions are made. Therefore the employer contribution is $5,600 and the employee can contribute up to $13,000 for 2019. In addition, Robert is 50 years old or older so he may make an additional catch-up contribution of $3,000. His total contribution is $5,600 + $13,000 + $3,000 = $21,600.

Which of the following qualified retirement plans are subject to mandatory minimum funding requirements? Defined benefit pension plans. Profit sharing plans. Money purchase pension plans. Target benefit plans. Section 401(k) plans. SIMPLE IRA.

The key here is to look for pension plans as all pension plans require minimum funding.

Maximum Performance, Inc.'s defined contribution plan has been determined to be top heavy. Which one of the following statements is NOT a requirement that applies to the plan? The employer must contribute a minimum of 3% of compensation or the contribution rate of the key employees (whichever is lower) per year to non-excludable, non-key employees for each year that the plan is top heavy. If the employer contribution to key employees is 2%, then the employer contribution to non-excludable, non-key employees must be 2%. The plan must use a vesting schedule that does not exceed either a 2-year cliff or 6-year graded vesting schedule. The plan must fully vest after three years of service if the vesting at two years is zero.

The vesting rules are a 3-year cliff or 2-6 year graded vesting schedule. (Hint: Answer "C" and "D" contradict each other, so it has to be one of these. If you picked up on that, you can easily narrow the questions down to 50/50.)

Which of the following tasks are the primary responsibilities of a plan trustee? Determining which employees are eligible for participation in the plan, vesting schedule, and plan benefits. Preparing, distributing, and filing reports and records as required by ERISA. Investing the plan assets in a "prudent" manner. Monitoring and reviewing the performance of plan assets.

There is a difference between the plan administrator and plan trustee. The duties explained in Statements "I" and "II" are responsibilities of the plan administrator.

Calculate the maximum contribution for an employee, age 41, earning $140,000 annually, working in a company with the following retirement plans: a 401(k) with no employer match and a money-purchase pension plan with an employer contribution equal to 12% of salary

This question is tricky. Notice it says maximum contribution - doesn't specify if its the employee or employer contribution. If it doesn't specify, assume both. The maximum 401(k) plan contribution is $19,000 (2019). The 12% money-purchase plan will add $16,800 ($140,000 × .12). So, adding the $19,000 and the $16,800 gives the correct answer of $35,800.

Eric works for Carpets, Inc. Carpets, Inc issued him both ISOs and NQSOs during the current year. Which of the following would be the most compelling reason why they might issue both ISOs and NQSOs? They want to issue over $80,000 in options that are exercisable in the same year. The NQSOs and ISOs are exercisable in different years. The company wants to provide the NQSOs to assist the individual in purchasing the ISOs. Since they are virtually the same there is no compelling reason to issue both in the same year.

When both ISOs and NQSOs are available in the same year the individual can exercise and sell the unfavored NQSOs to generate enough cash to purchase and hold the favored ISOs. It would also be valuable to have both if they issued over $100,000 in options exercisable in the same year because there is a $100,000 limit on ISOs.

Hot Dog Moving Company (HDM) sponsors a 401(k) profit sharing plan. In the current year, HDM contributed 20% of each employee's compensation to the profit sharing plan. The ADP of the 401(k) plan for the NHC is 2.5%. Alex, who is age 57, earns $177,778 and owns 7.5% of the company stock. What is the maximum amount that he may defer into the 401(k) plan for this year? $8,000 $11,000 $14,000 $16,500

You should probably know the ADP test requirement contribution chart. 0-2% -> 2x ADP for NHC 2-8% -> 2% + ADP for NHC 8% and over -> 1.25x ADP for NHC Also this question is about more than calculating the amount he can contribute as a HCE, its also about him being 50+ which means he can contribute more as a catch up. Alex is highly compensated because he is more than a 5% owner, so the maximum that he can defer to satisfy the ADP Test requirements is 4.5% (2.5% + 2%) and because he is over 50, he can defer the additional $6,000 (2019) as a catch-up contribution. Alex can defer $8,000 (4.5% × $177,778) and $6,000 (the catch-up) for a total of $14,000.

The Health Insurance Portability and Accountability Act of 1996 (HIPAA) impacts an employee and employer in which of the following ways: An employee without creditable coverage can generally only be excluded by the group health insurance plan (if offered) for up to twelve months. The waiting period is reduced by the amount of "creditable coverage" at a previous employer. If the employee does not enroll in the group health insurance plan at the first opportunity, an 18-month exclusion period may apply.

all of the above are true

Which one of the following is a possible disadvantage of a Simplified Employee Pension plan (SEP) for an employer? The SEP's trustee is subject to ERISA's prohibited transaction excise tax penalties. A SEP must have a fixed contribution formula that is non-discriminatory. SEPs prohibit forfeitures. Employer contributions to a SEP are subject to payroll taxes.

question is trying to have you tease out the disadvantage of SEP vs other qualified plan. Options "A" and "D" are false. Option "B" is a mandatory characteristic of all qualified defined contribution pension plans but not profit sharing plans.

Which of the following accurately describes a 403(b) plan? A 403(b) plan is a noncontributory qualified profit sharing plan. Because of catch-up provisions, the investment risk of the assets within a 403(b) plan is borne by the plan sponsor not the participant. A participant's benefits within a 403(b) plan will generally vest according to a 3 to 7 year graduated vesting schedule, however, a 5-year cliff vesting schedule may be used. 403(b) plan assets can be invested indirectly in stocks and bonds through annuities or mutual funds.

403B is not a qualified plan. Its considered an employee deferral plan.

Which of the following statements concerning rabbi trusts is correct? A rabbi trust generally makes distributions to the executive to pay for income tax attributable to the earnings within the trust. A rabbi trust calls for an irrevocable contribution from the employer to finance promises under a nonqualified plan, and funds held within the trust cannot be reached by the employer's creditors. A rabbi trust can only be established by a religious organization exempt from tax under IRC 501(c)(3). A rabbi trust is established to avoid constructive receipt.

Option a is not correct and more common in a secular trust. Option b describes a secular trust. Option c is a false statement.

Which statement(s) accurately reflect(s) the Tax-Sheltered Annuity (TSA) provisions: Salary reductions into a TSA are exempt from all payroll taxes. The annual elective deferral limit may be increased by up to $3,000 for employees of certain organizations who have completed 15 years of service and meet certain other requirements. Tax sheltered annuities must allow participants to invest in mutual fund, annuities and/or fixed income securities. To calculate the maximum exclusion allowance for make-up calculation purposes, the participant's years of service and the amount of total excludable contributions made in the prior three years are needed.

Statement "I" is incorrect because deferrals are still subject to Social Security and Medicare taxes. Statement "III" is incorrect because TSAs can only invest in mutual funds or annuities and not any direct investments. Statement "IV" is incorrect because the total excludable contributions must be for all prior years, not just the past three.

Which of the following statements concerning rabbi trusts is (are) CORRECT? A rabbi trust is a trust established and sometimes funded by the employer that is subject to the claims of the employer's creditors, but any funds in the trust cannot generally be used by or revert back to the employer. A rabbi trust calls for an irrevocable contribution from the employer to finance benefits promised under a nonqualified plan, and funds held within the trust cannot be reached by the employer's creditors. A rabbi trust may not be held off-shore as a result of the American Jobs Creation Act of 2004. The American Jobs Creation Act of 2004 prohibits "springing irrevocability" for a rabbi trust if there is a change of control or ownership.

Statement "II" describes a secular trust. Statement "IV" is incorrect because AJCA 2004 does allow springing irrevocability in these circumstances, but not for bankruptcy.

A SEP-IRA is a form of defined contribution plan (although not a qualified plan). Which of the following apply to BOTH the SEP-IRA and a traditional defined contribution plan? Employer deductions limited to 15% of covered payroll. Requires a definite, written, non-discriminatory contribution allocation formula. Contributions cannot discriminate in favor of highly compensated employees. Employer contributions subject to Medicare and Social Security taxes. Affiliated service group rules apply. Top-heavy rules do NOT apply. Permissible disparity or integration is NOT allowed. I, II, VI, and VII only. II, III, IV and VI only. II, III and V only. I, II, III, V and VII only.

Theres a lot to this question. I don't understand a lot of the options here so if its a scenario where you're unsure don't get unnerved. Start to figure out what you know. I know that I is wrong because its limited to 25% and not 15%. I know II is correct. Knowing those two allows you to rule out a few options. For sure its not A & D because they both contain I. Leaving B & C, which both contain II so moving onto choice III, they both contain so its irrelevant. The difference between B & C is B has IV and VI as a choice so its good to determine if any of those are ruled out by something you know. I know that VI is incorrect as top heavy rules do apply since DC plans do not like to favor top heavy. Best educated guess is C because you're attempting to rule out the other ones as wrong or higher probability of being wrong. Defined contribution plans have an employer deductibility limit of 25% of covered payroll. All defined contribution plans must have a written allocation formula so assets can be distributed in the mandated individual accounts. Employer contributions must bear uniform resemblance to compensation and cannot discriminate in favor of highly compensated. Employer contributions are not subject to any payroll related taxes. Top-heavy rules do apply to both. Both plans can integrate with Social Security (sometimes called permissible disparity). (Note: 5305-SEP does not allow permissible disparity.)

Target benefit plans and defined benefit plans have which of the following characteristics in common? Minimum funding standards apply. Qualified joint and survivor annuity requirements apply. High investment earnings increase participant retirement benefits. The employer is obligated to provide a specified benefit in retirement.

1) need to determine what kind of pension plan it is. Target benefit is a defined contribution pension plan while as a defined benefit plan is a defined benefit pension plan Both plans are pension plans, therefore Statements "I" and "II" apply. Statement "III" applies only to the target benefit plan (because it is a DC plan) NOT the DB plan. Statement "IV" applies only to the DB plan NOT the target benefit plan.

Jack is a greeter at Doogle Mart and works full time. Unfortunately, he does not own any of the stock as the business has increased significantly over the years. He turned 70 ½ on November 19th of year 2. The table below reflects the balance in each of his retirement accounts at the end of each specified year. He no longer works for Walton Company. Assume the life expectancy factor for ages 70, 71, 72, and 73 are 27.4, 26.5, 25.6 and 24.7 respectively. What is the minimum distribution for Jack for year 2, exclusively?

17,336 Minimum distributions must be taken by Jack for all traditional IRAs and all qualified plans other than Doogle Mart, since he is still employed there he can defer until age 73. His attained age at the end of year 2 is still 70 because he will not turn 71 until May of year 3, which equates to a factor of 27.4. The total balances from the end of year 1 total $475,000, which is $1 million less the balance for the Roth IRA and the $400,000 in the Doogle Mart. $475,000/27.4=17,336. Roth 401k subject to RMD's

Which of the following individuals are "key employees" as defined by the Internal Revenue Code? A more-than-5% owner of the employer business. An employee who received compensation of more than $120,000 from the employer. An officer of the employer who received compensation of more than $180,000. A 1% owner of the employer business having annual compensation from the employer of more than $55,000.

A key employee is an individual who (1) owns more than 5% of the business, (2) is an officer with compensation greater than $180,000 (2019), or (3) owns greater than 1% of the business and has compensation greater than $150,000. I. and III. are defined key employees. II. defines a highly compensated employee, not a key employee. IV. should state that compensation was more than $150,000.

What is a qualified plan?

A plan created under the section 401. Probably why 457 and 403b not considered qualified?

Which of the following plans is subject to PBGC coverage? A cash balance plan for a local store selling nutrition supplements and drinks. A target benefit plan for a publicly traded company. An integrated defined benefit plan for an architect's office that employees two owners, who are architects, three junior architects and one administrative assistant. A new comparability plan with three groups for a firm with 26 employees.

All DB plans (DB and cash balance) must be covered by PBGC except for professional firms with 25 or fewer employees. Choice b is not correct because it is defined contribution plan and is not subject to PBGC. Choice c is not correct because although it is defined benefit plan, it is not subject to PBGC because it is a professional firm with fewer than 25 employees. Choice d is not correct because it is defined contribution plan and is not subject to PBGC.

Services provided on a discounted or free basis to employees are not includible in taxable income to the employee under which of the following circumstances? The employer must incur no substantial cost in providing the service. Services offered to the employees must be in the line of business in which they are working. Services cannot be discounted more than 25% of the price that is available to customers. If there is a reciprocity agreement between two unrelated employers in the same line of business.

All statements are correct except for Statement "III". This is because the percentage of discount that is stated is limited to 20%.

Your client, a single-filer, has an income of $80,000. Which of the following conditions would prevent a deductible IRA contribution from being made by your client? Participated in a Section 457 deferred compensation plan. No other retirement plans were available to the employee. Made contributions to a 403(b) plan. Received retirement payments from a pension plan at age 65 (no longer an employee at the sponsoring employer). Has account in previous employer's profit-sharing plan. Received no employer contributions. No forfeiture allocations were made. Eligible to participate in a defined benefit plan, but waived participation when it was calculated that employee retirement benefits would be greater with the IRA. I, II and IV only. II, IV and V only. II, III, IV and V only. II and V only.

An active participant is an employee who has benefited under one of the following plans through a contribution or an accrued benefit during the year: qualified plan; annuity plan; tax sheltered annuity (403(b) plan); certain government plans (does not included 457 plans); SEPs; or SIMPLEs. Statement I is a non-qualified deferred comp plan (not one of the plans listed above) and therefore not to be taken into consideration for active participation status. Statement II & V are on the list above. For a defined benefit plan, an individual who is eligible for the plan is automatically considered an active participant. Statement "III" is not active participation, rather it is retirement, and Statement "IV" as described without contributions or forfeitures is not "active participation," but a change in conditions regarding employer contributions or forfeitures could stem deductibility of IRA contributions.

Eldrick, age 40, established a Roth IRA 3 years ago and was tragically struck and killed by an errant golf ball hit by a drunk spectator at a golf tournament. Eldrick had contributed a total of $10,000 to the account and had converted $20,000 from his traditional IRA. His 20 year old son, Charlie, inherited the Roth IRA, which now has a balance of $60,000. Which of the following statements is correct? Charlie can distribute the entire balance from the Roth IRA without it being subject to any income tax or penalty the month after Eldrick dies. Charlie must take minimum distributions from the Roth IRA the year Eldrick dies. If Charlie begins taking minimum distributions, then the first distribution will be partially taxable. Charlie could delay taking a distribution from the Roth IRA for several years and avoid all penalties and income tax on the distribution.

Answer a is incorrect because the distribution would not be a qualified distribution since the five year rule has not been met. Answer b is not correct as he could begin taking distribution the year following death or take a full distribution within five years. Answer c is not correct, because the first distribution would consist of previously taxed contributions and would therefore not be taxable. Answer d is correct as he could delay taking a distribution from the account for two years. At that point, the distribution would be qualified since it meets the five-year rule and is on account of death. The distribution would avoid all income tax and penalties.

Jacque's wife just lost her job and they had a death in the family. Jacque is planning on taking a hardship withdrawal from his 401(k) plan to pay for living expenses and funeral costs. Which of the following is correct regarding hardship withdrawals? Hardship withdrawals can be taken even if there is another source of funds that the taxpayer could use to pay for the hardship. Hardship withdrawals are beneficial because although they are taxable, they are not subject to the early withdrawal penalty. Hardship withdrawals can be taken from elective deferral amounts or vested employer contributions. Unless the employer has actual knowledge to the contrary, the employer may rely on the written representation of the employee to satisfy the need of heavy financial need.

Answer a is not correct as there must not be another source of funds. Answer b is not correct as they are generally subject to a penalty unless there is an exception under IRC 72(t). Answer c is not correct as a hardship distribution can only be taken from employee deferrals.

Which of the following is true concerning IRA contributions? An employee who makes voluntary contributions to a 401(k) plan is not considered an active participant. An employee who receives no contributions or forfeiture allocations in their employer's profit sharing plan is not considered an active participant. An employee who makes no voluntary contributions to a thrift plan yet receives forfeiture allocations to a profit sharing plan is not considered an active participant. An employee participating in a Section 457 plan is considered an active participant if employee pretax deferrals are elected.

Answers "A" and "C" are conditions of being considered an active participant. Answer "D" is incorrect because 457 plan participants are not considered active participants for IRA contribution purposes. Theres a difference between being covered and being active.

Which of the following statements concerning cash balance pension plans is correct? The cash balance plan is a defined benefit plan because the annual contribution is defined by the plan as a percentage of employee compensation. The cash balance plan provides a guaranteed annual investment return to participant's account balances that can be fixed or variable and is 100% guaranteed by the Pension Benefit Guarantee Corporation. The cash balance plan uses the same vesting schedules as traditional defined benefit plans. The adoption of a cash balance plan is generally motivated by two factors: selecting a benefit design that employees can more easily understand than a traditional defined benefit plan, and as a plan that has more predictable costs associated with its funding.

Cash balance plans are defined benefit plans due to the guaranteed investment returns and benefit formula, not simply a contribution amount. While cash balance plans provide guaranteed rates of return, they are not 100% guaranteed by the PBGC (PBGC has coverage limits). Cash balance plans use 3-year cliff vesting only. Choice d is correct.

Retirement plans qualified under IRC Section 401(a) have many benefits for employers and employees. Which of the following is correct regarding qualified plans? All employer contributions to a qualified plan are fully deductible in the year of contribution. Payroll taxes are avoided for all contributions to a qualified plan. All qualified plan assets are held in a tax exempt trust and all earnings within the trust are deferred from taxation until distributed from the plan. The non-alienation of benefits rule under ERISA provides complete protection from all creditors, including the IRS, unless the funds are distributed from the plan.

Choice a is incorrect because there are limits to the deductibility of contributions to a qualified plan. Generally, only contributions up to 25% of covered compensation can be deducted for a year. Choice b is incorrect as employee contributions are subject to payroll tax. Choice d is incorrect as the IRS can get to assets in a qualified plan as well as spouses via a QDRO

Which of the following statements regarding ISOs and NQSOs is correct? The income tax treatment of a cashless exercise of an ISO is favorable compared to the cashless exercise of a NQSO. One of the disadvantages of an ISO is that the sale of the stock attributable to an ISO may result in the taxpayer paying alternative minimum tax. IRC 409A provides harsh penalties when a company grants an ISO or NQSO with a strike price that exceeds the current FMV of the employer's tax. To the extent that the aggregate fair market value of stock with respect to which incentive stock options are exercisable for the 1st time by any individual during any calendar year exceeds $100,000, such options shall be treated as options which are not incentive stock options.

Choice a is not correct as the tax treatment is the same for a cashless exercise of an ISO and an NQSO. Choice b is not correct. The sale of an ISO share of stock will generally have a negative adjustment for AMT, not positive, and therefore, it would not result in AMT. Choice c is not correct as 409A would apply if the strike price was less than the FMV on the date of grant.

Wilber receives incentive stock options (ISOs) with an exercise price equal to the FMV at the date of the grant of $15. Wilber exercises these options 3 years from the date of the grant when the FMV of the stock is $35. Wilber then sells the stock 3 years after exercising for $45. Which of the following statements is (are) true? At the date of grant, Wilber will have ordinary income equal to $15. At the date of exercise, Wilber will have W-2 income of $20. At the date of sale, Wilber will have long-term capital gain of $30. Wilber's employer will have an income tax deduction related to the exercise of the option by Wilber.

Choice a is not correct as there is no income at the date of grant because the strike price equals the FMV. Choice b is not correct as there is no regular tax for ISOs. Choice d is not correct because the employer will not have an income tax deduction. Grant 15 Exercise @35 (AMT adjustment of $20) Sale $45 $10 LTCG from gain (exercise to sale) $20 LTCG from exercise Total $30 LTCG More information can be found in the infographics, Getting Started>Additional Resources>Infographics.

Kipton is an executive with BigRock. As part of his compensation, he receives 10,000 shares of restricted stock today worth $20 per share. The shares vest two years from today, at which point the stock is worth $30 per share. The vesting schedule is a 2-year cliff schedule. Kipton holds the stock for an additional 18 months and sells at $45 per share. Which of the following is correct? The grant of stock is taxable to Kipton today. The value of the shares is taxable to Kipton when the stock vests. If Kipton were to make an 83(b) election, he would have converted $30 of the gain from ordinary to capital. When Kipton sells the stock for $45 per share, his basis is $30 regardless of whether he files an 83(b) election.

Choice a is not correct because the stock is forfeitable. Choice c is not correct because it would have converted $10, not $30. Choice d is not correct, because the basis would be different.

Match the following statement with the type of retirement plan which it most completely describes: "A plan which requires annual employer contributions equal to a formula determined by each participant's salary" is a... Profit sharing plan. Money purchase plan. SIMPLE IRA. Defined benefit plan.

Defined benefit plan and cash balance plan (Answer "D") contributions are determined by age, as well as salary. Answer "A" doesn't require annual contributions. Answer "C" has employer contributions determined by the amount of employee deferrals.

Which of the following statements regarding determination letters for qualified plans is true? When a qualified plan is created, the plan sponsor must request a determination letter from the IRS. An employer who adopts a prototype plan must request a determination letter from the IRS. If a qualified plan is amended, the plan sponsor must request a determination letter from the Department of Labor. A qualified plan which receives a favorable determination letter from the IRS may still be disqualified at a later date.

Determination letters are issued by the IRS at the request of the plan sponsor. The plan sponsor is not required to request a determination letter. Even if the determination letter is requested and approved, the IRS may still disqualify the plan.

Generally, which of the following are noncontributory plans? 401(k) and money purchase pension plans . 401(k) and thrift plans. Thrift plans and ESOPs. Money purchase pension plans and profit sharing plans.

Employers generally contribute to Money Purchase Pension Plans, ESOPs, and Profit Sharing Plans. Employees contribute (thus contributory plans) to 401(k)s and Thrift Plans. Its only the last option because the 3rd option includes thrift plans. If just ESOP then that would also be an answer

Financial Training Team (FTT) develops training materials for finance professionals across the country. Chad, who just turned age 62, owns 15% of FTT and earns $200,000 per year and is a participant in his employer's 401(k) plan, which includes a qualified automatic contribution arrangement and the associated mandatory non-elective contribution. The actual deferral percentage test for the non-highly compensated employees is 2.5 percent. FTT made a 20% profit sharing plan contribution during the year to Chad's account. What is the maximum amount that Chad can defer in the 401(k) plan during 2019?

Even if ADP were an issue here, you still need to check to see the maximum annual contribution - employer contribution to see if you can defer what you thought you could. Under ADP he'd be able to defer $13K (2.5% +4%) and age 50+ contribution. Total would have been 19K, but 19K + 40K employer contribution exceeds $56k. They gave a lot of excess info on this to trick you. Having qualified automatic contribution arrangement should allow you to bypass ADP testing. The 401(k) plan avoids ADP testing because it is a QACA. Therefore, the ADP for the NHCE is irrelevant. However, the max that can be contributed is limited by IRC 415(c). The employer is contributing $40,000 to the profit sharing plan plus $6,000 as a non-elective contribution (3% of $200,000). Since the 2019 limit is $56,000, Chad can only contribute $10,000 plus the catch-up contribution of $6,000.

Which statement(s) is/are true regarding qualified profit sharing plans? A company must show a profit in order to make a contribution for a given year. A profit sharing plan is a type of retirement plan and thus is subject to minimum funding standards. Forfeitures in profit sharing plans must be credited against future years' contributions. Profit sharing plans should make contributions that are "substantial and recurring." IV only. I and III only. I, II and IV only. I, II, III and IV.

Even though not mandatory, regulations require a profit sharing plan to make substantial and recurring contributions. No profits or retained earnings are required to make a profit sharing contribution. Minimum funding requirements apply to pension plans, not profit sharing plans. Forfeitures in profit sharing plans are usually allocated to the remaining participants' individual accounts.

Corey is covered under his employer's Profit-Sharing Plan. He currently earns $500,000 per year. The plan is top heavy. The employer made a 5% contribution on behalf of all employees. What is the company's contribution for him?

For a profit sharing plan the contribution is limited to the lesser of $56,000 (2019) or covered compensation. In this case the contribution will be limited by the covered compensation limit of $280,000. $280,000 × 5% = 14,000. The fact that the plan is top heavy is irrelevant since all employees are receiving a contribution greater than 3%. This is most likely considered a safe harbor plan due to 3% contribution

Spenser is covered under his employer's top heavy New Comparability Plan. The plan classifies employees into one of three categories: 1) Owners, 2) Full-time employees, 3) Part-time employees. Assume the IRS has approved the plan and does not consider it to be discriminatory. The employer made a 4% contribution on behalf of all owners, 2% contribution on behalf of all Full-time employees and 1% contribution on behalf of all part time employees. If Spenser currently earns $50,000 per year and is a full-time employee, what is the contribution that should be made for him?

For a profit sharing plan the contribution is limited to the lesser of $56,000 (2019) or covered compensation. Since the plan is top heavy, the plan must provide a benefit to all non-key employees of at least 3%, therefore; 50,000 × 3% = $1,500.

Which of the following accurately describes a qualified group life insurance plan? The plan must benefit 70% of all employees, or a group consisting of 85% non-key employees, or a non-discriminatory class, or meet the non-discrimination rules of Section 125. Employees who can be excluded are: those with fewer than 3 years service, part-time / seasonal, non-resident aliens, or those covered under a collective bargaining unit. A non-discriminatory classification is one which has a bottom tier with benefits no less than 10% of the top tier and no more than 200% increase between tiers. The minimum group size is 10.

For statement III to be a correct choice, it should state: A qualified group life insurance plan, if using a non-discriminatory classification, will have a bottom tier with benefits no less than 10% of the top tier and no more than 250% increase between tiers.

Bob Thornton received his NonQualified Stock Option (NQSO) from his company (publicly traded on the NYSE), one year ago last week, with an option exercise price and stock price of $30. He told you recently, as his trusted financial adviser, that he desperately needed cash, and so he exercised the options last week on the one year anniversary with the stock price at $40 per share, and he sold the stock as it climbed to $45 per share. What will the tax ramifications of these transactions be? Bob will be taxed at capital gain rates only when the option is exercised and again for the difference when the stock is sold. Bob will be taxed as W-2 income for the fair market value of the option less the exercised price when it is exercised, and then he will be taxed at capital gain rates for the balance when the stock then subsequently sold (long or short term). Bob will be taxed at ordinary income rates on the difference between the exercise price and the fair market value of the stock when the option is granted and then on capital gain rates when the stock is sold for the fair market value. There are no consequences to this circumstance until Bob sells the stock with all proceeds being taxed at capital gains rates.

He will be required to report the gain as W-2 income of $10 per share when the option is exercised and then another $5 capital gain (long or short) when sold as the question indicates.

Jan is an executive at Papers Unlimited. As part of her compensation she has a restricted stock plan that allows her to receive 500 shares of stock after she completes 5 years of service. At the time of grant (three years ago) the stock was trading at $5 per share. The stock is currently trading at $25 per share and she has been with the company for 3 years. Which of the following is true? If she made the 83b election today she would recognize W-2 income of $12,500. If she made the 83b election at the time of grant and she left the company today she would recognize capital gain of $10,000. If she made the 83b election at the time of grant and she left the company today she would recognize a loss of $2,500. If she made the 83b election at the time of grant and 5 years later sold the stock for $35 per share her capital gain treatment would be $15,000.

If she made the 83b election at the time of grant then she would have had W-2 income at the time of $2,500 (500 × $5). If she later sold for $35 per share then she would recognize capital gain of $15,000 (500 × ($35-$5)). Note that by saying she sold the stock the question implies that she met the vesting requirement. If she left the company today before meeting the vesting period she would not be allowed to take a loss on W-2 income that she included in income in the year of grant. The 83b election cannot be made today - it must have been made 30 days after the date the stock was initially transferred at grant.

David is awarded an immediately vested, non-qualified stock option for 1,000 shares of company stock with an exercise price of $35 per share while the stock price is currently $33 per share. What are the tax ramifications, if any at the date of the grant? $0 The gain between exercise and actual price of $2,000 is immediately taxable. The awarded option price value of $33,000 will be immediately taxable. Because these are unrealized gains, neither the option value nor the gain are taxable until the stock is finally sold.

In the case of NQS Options, the option is not taxed at the grant if the exercise price is equal to or greater than the fair market value of the stock.

If a stock option is vested when it is received, and has a readily ascertainable value it is: Assigned that value for taxation purposes. Taxable when the stock is sold. Taxable as soon as it is exercised. Immediately taxable.

It looks like compensation if fully vested and below market value. Vested options are taxable based on the value of the option to the extent the Fair Market Value exceeds the option price.

Which of the following are legal requirements for 401(k) plans? Employer contributions do not have to be made from profits. Employee elective deferral elections must be made before the compensation is earned. Hardship rules allow in-service withdrawals which are not subject to the 10% early withdrawal penalty. ADP tests can be avoided using special safe harbor provisions. I, II and IV only. I, III and IV only. II, III and IV only. I, II, III and IV.

Look closely at this one. 1 and 4 are straight forward, but 2 is saying that the employee needs to have made the "deferral election" before income is made. This doesn't mean they need to contribute the funds before income is earned. Hardship withdrawals are considered premature withdrawals and are subject to income tax and the 10% early withdrawal penalty if the employee is under age 59 1/2.

Which of the following retirement plans would permit an employee (filing single status) making $100,000 a year to still make a fully deductible $6,000 contribution to an IRA in 2019? 401(k) 403(b) SEP 457

Look for the differences here - look for the NQL plan. IRC Section 457 plans are nonqualified deferred compensation plan, and therefore do not make the employee an "active" participant in a qualified retirement plan. The 401(k) is a qualified plan and the 403(b) and SEP are 'wannabe' be plans that would make the employee an "active" participant.

A hybrid plan that uses a discretionary contribution but adjusts for age is a form of a: Profit sharing plan. Money purchase plan. Cash balance plan. Defined benefit plan.

Look for the word discretionary. The other ones are not. Answers "B," "C" and "D" all require minimum contribution levels. Answer "A" - Profit sharing plan only requires that contributions be "substantial and recurring." More specifically, an age-based profit sharing plan would be correct.

A qualified money purchase pension plan contribution (by the employer) is influenced by which of the following factors: Total return on portfolio assets. Forfeitures from non-vested amounts of terminated employee accounts. Increases in participants' compensation due to inflation or performance-based bonuses. Minimum funding as determined by an actuary. I and II only. II and III only. III and IV only. II, III and IV only.

MPPP is a type of pension plan, but it is considered a Defined Contribution Pension plan (Not to be confused with defined contribution plans - those are notated as defined contribution profit sharing plans) Not all pension plans need actuaries to determine amount to put in. The DC Pension plans do not while as the DB pension plans do require. Forfeitures may reduce employer contributions due to contribution offsets or Section 415 limitation on annual additions. Increased compensation will result in increased contributions by the employer, subject to Section 415 limitations. Returns on portfolio assets and actuary funding are a concern in defined benefit plans. A money purchase plan is defined contribution plan.

Highly compensated employee

May also include top 20% of paid employees

David Lee, age 63, was a participant in a stock bonus plan sponsored by VH, Inc., a closely held corporation. David's account was credited with contributions in shares of VH stock to the stock bonus plan and VH Inc. deducted $80,000 over his career at VH. The value of the stock in the account today is worth $1 million. David takes a distribution (year 1) of one-half of the VH stock in his stock bonus plan account valued at a fair market value of $500,000. If David sells the stock for $600,000 nine months after receiving the distribution (year 2), then which of the following statements are true? David will have ordinary income of $80,000 in year 1 and capital gain of $520,000 in year 2. David will have ordinary income of $40,000 in year 1 and capital gain of $560,000 in year 2. David will have ordinary income of $460,000 in year 1 and capital gain of $100,000 in year 2. David will have ordinary income of $500,000 in year 1 and capital gain of $100,000 in year 2.

NUA treatment is not applicable because David did not take a lump sum distribution of stock from the plan. Therefore, the distribution is treated as ordinary income 500K Ordinary Income / 100k cap gain in year 2

Brisco, age 51, is the Executive VP of sales at Doggie Daycare (DD). His base salary is $300,000 with a potential bonus of 50%. Brisco is a participant in his employer's 401(k) plan and always defers the maximum amount. The DD 401(k) plan has the following features and characteristics: Includes a Roth account that is not a safe harbor plan, but has a 50% match up to 4%. The ADP for the NHC is 4.5%. The plan has $3 million in assets that are managed by two asset management firms. DD also sponsors a defined benefit plan that provides a benefit based on years of service and final salary. The DD DB plan provides for 1.5 percent per year of service for the first 20 years and 2 percent for years above 20, up to a max of 35 years. On the weekends, Brisco paints murals. His entity, Wall Works LLC (WW), is a single owned LLC taxed as a disregarded entity. Brisco would like to establish a retirement plan for the income that he earns in WW. He expects to earn $60,000 ever year in WW and wants to know what the best retirement plan is for his business. Which plan would you recommend for him? SIMPLE Defined Benefit Plan 401(k) plan SEP

Neither a SIMPLE nor a 401(k) plan will work because he is already deferring $18,200 (6.5% (4.5% + 2%) times $280,000) to his 401(k) plan. Therefore, the choices are a 412(e) plan, which is a defined benefit plan funded with insurance products or a SEP, which is extremely easy to set up and one that he can contribute around $11,000 to annually.

Beth works for MG Inc. and was hired right out of school after graduating with a double major in marketing and advertising four years ago. Beth receives a $12,000 distribution from her designated Roth account in her employer's 401(k) plan as a result of her being disabled. Immediately prior to the distribution, the account consisted of $15,000 of investment in the contract (i.e., designated Roth contributions) and $5,000 of income. What are the tax consequences of this distribution? She will have $12,000 of income. She will have $5,000 of income. She will have $3,000 of income. She will have no income tax consequences resulting from the distribution.

Non qualified distributions from a Roth account are subject to tax on a prorata basis. Since 75% of the value in the account consists of basis and the remaining 25% consists of earnings, that same ratio of basis to income will apply to the $12,000 distribution. It is not a qualified distirubiton because she has not held the account for at least five years.

Kyle had contributed $20,000 in nondeductible contributions to his traditional IRA over the years. This year the account balance was $52,000 and he made a withdrawal of $5,000. What amount is reported on Kyle's Form 1040? $5,000 only $1,923 only $3,077 only Both $5,000 and $3,077

On Form 1040 Kyle will report the total distribution of $5,000 and the taxable amount of the distribution of $3,077 calculated as $32,000 ÷ $52,000 × $5,000. account balance = $52,000 non-deductible contributions = $20,000 (not taxed at distribution) $32,000 would be taxable. Because there is both taxable and non-taxable money, each distribution is a pro-rata distribution of both. 32k/52k = .6154 5,000 × 61.54% = 3,076.92

Which of the following is NOT a qualified employee fringe benefit? Medical expenses NOT covered by medical health plan are paid under a reimubursement plan. A major medical health insurance plan or HMO premiums. Long-term disability insurance. A $150,000 group term life insurance policy.

Only $50,000 of group term life is a qualified benefit. Amounts of term life insurance in excess of $50,000 is taxable to employee using Section 79, Table 1.

Which of the following is false regarding incentive stock options? No regular taxable income will be recognized by the employee when the qualified option is granted or exercised. The income from sale of the qualified option will always be taxed as capital gains when the stock is sold. The income from sale of the qualified option will be taxed as ordinary income regardless of when the stock is sold. The employer will not be able to deduct the bargain element of the option as an expense under any circumstance. For favorable tax treatment the stock must be held two years from grant and one year after exercise. II and IV only. II, III and IV only. III only. I, II and V only.

Only Statements "I" and "V" are true. The rest are all false. In Statement "II", be careful of "always"! In Statement "III," if held longer than one year, they receive capital gains treatment. In Statement "IV," the bargain element will be deductible if the sale is a disqualifying disposition.

A premature distribution from a qualified retirement plan is allowed at age 52 without a 10% penalty tax when a participant: Becomes obligated for payment of plan benefits to an alternate payer under a qualified domestic relations order (QDRO). Separates from service and takes an accepted form of systematic payment. Remains with current employer but elects to take systematic payments over the life of the participant and spouse. I only. III only. I and II only. I, II and III.

Only the first two are correct. The last one is not an annuity despite making it seem so. Distributions under a QDRO are not taxable to the taxpayer actually making the disbursement from his/her account. IRC 72(t) allows Substantially Equal Payment Plans (SEPP) to escape the 10% penalty as long as the payments continue for the longer of 5 years or until age 59 1/2. No in-service withdrawals are exempted from the 10% early withdrawal penalty.

Which of the following is true concerning 401(k) plans regarding Highly Compensated (HC) employees and Non-Highly Compensated (NHC) employees? Non-discriminatory rules state at least 80% of all NHCs must be covered by the plan, or the ratio of NHCs covered by the plan must be at least 80% of the HCs, or the average benefit percentage for NHCs is at least 80% of the benefit percentage of the HCs. The Actual Deferral Percentage (ADP) Test limits employee voluntary contributions to the plan for the HCs. To accurately calculate the ADP, the administrator needs to know the percentage of income the NHCs contribute to the plan and how much the HCs defer to the plan. The Actual Deferral (ADP) Test takes all employees into account when calculating the test. The average ratio of HCs may not exceed 125% of the ADP of the NHC group if the ADP of the NHC group is 6% or more.

Option "A" would be correct if the percentages referenced were "70%" rather than "80%." Option "C" is incorrect because only employees who are eligible to participate are included in the ADP test. Option "D" - At a NHC ADP level of 6%, the HC may not exceed 8% (6% + 2%). The 125% category does not start until NHC ADP level equals 8% or higher.

How is life insurance utilized to finance the obligation of an employer under a non-qualified deferred compensation plan? A company can defer compensation that would otherwise be due an employee and allow the employee to use this money to purchase life insurance listing himself as the owner of the policy. A company can defer future compensation that will be due an employee and use the amount to purchase life insurance in the employee's name while the company pays premiums for the policy. A company can defer compensation that would otherwise be due an employee and use the amount to purchase life insurance on the employee in the company's own name while paying the premiums for the policy. A company can defer compensation that otherwise would not yet be due an employee and use this projected amount of money to purchase life insurance in the employee's name while the company pays premiums for the policy. Solution: The correct answer is C.

Option "C" is the only answer which describes the workings of a deferred compensation package accurately. Option "A" is incorrect because the employee does NOT purchase the insurance. Option "B" is incorrect because it is compensation due NOW, not "future compensation." Option "D" is incorrect because the deferral is not "projected income."

Which statements accurately reflect the provisions for a self-employed owner (partnerships and sole proprietorships) in a small business pension plan? Loans are available to owners and employees alike, if each has equal right and terms of the loans. Contributions for owners are based on net earnings rather than wages. Contributions for employees (as percentage of salary) is the same as for the self-employed owner (as a percentage of profit). Lump-sum distribution tax treatment allowed for employees, but not for owners, except in the case of disability.

Owners must do a conversion [EE contr rate ÷ (1+ EE contr rate)] e.g., .15 ÷ (1+.15) = .13043 so owner's contribution as a percentage of profits is lower than the employees' percentage of wages earned. Loans available to all. Contributions are based on net earnings rather than wages for owners. Lump Sum distribution tax treatement allowed for employees, but not owners, except in case of disability.

Ernest converted his Traditional IRA to a Roth IRA on Dec 15, 2014. He was 35 years of age at the time and had never made a contribution to a Roth IRA. The conversion was in the amount of $60,000 ($10,000 of contributions and $50,000 of earnings). Over the years he has also made $15,000 in contributions. On May 15, 2018 he withdrew the entire account balance of $100,000 to pay for a 1 year trip around the world. Which of the following statements is true? $25,000 of the distribution will be subject to income tax and $85,000 of the distribution will be subject to the 10% early withdrawal penalty. $25,000 of the distribution will be subject to income tax and the 10% early withdrawal penalty. Some of the distribution will be taxable but the entire distribution will be subject to the 10% early withdrawal penalty. None of the distribution will be taxable nor will it be subject to the 10% early withdrawal penalty.

Roth distributions are tax free if they are made after 5 years and because of 1)Death, 2)Disability, 3) 59.5 years of age, and 4)First time home purchase. He does not meet the five year holding period or one of the exceptions. His distribution does not received tax free treatment. The treatment for a non-qualifying distribution allows the distributions to be made from basis first, then conversions, then earnings. His basis will be tax free. The conversion is also tax free since we paid tax at the time of the conversion on those earnings. The remaining earnings since establishment of the Roth are $25,000 (100,000 - $15,000 in basis - $60,000 in conversions) and will be taxed. The 10% penalty does apply to this distribution since he does not qualify for any of the exceptions to the penalty. The contributions escapes penalty but the conversions and earnings of $85,000 are subject to the 10% early withdrawal penalty. Remember that in order for the conversions to escape the 10% early withdrawal penalty the distribution must occur after a 5 year holding period beginning Jan 1 in the year of conversion or meet one of the 10% early withdrawal exceptions.

Mayu made a contribution to his Roth IRA on April 15, 2014 for 2013. This was his first contribution to a Roth IRA. Over the years he has made $20,000 in contributions. On May 15, 2019 he withdrew the entire account balance of $45,000 to pay for his daughter's college education expense. He is 55 years of age. Which of the following statements is true? He will not include anything in income and will not be subject to the 10% early withdrawal penalty. He will include $25,000 in income but will not be subject to the 10% early withdrawal penalty. He will include $25,000 in income and will be subject to the 10% early withdrawal penalty on $25,000. He will include $45,000 in income and will be subject to the 10% early withdrawal penalty on $45,000.

Roth distributions are tax free if they are made after 5 years and because of 1)Death, 2)Disability, 3) 59.5 years of age, or 4)First time home purchase. Although he met the 5 year rule, he did not meet one of the four qualifying reasons. His distribution does not received tax free treatment. The treatment for a non-qualifying distribution allows the distributions to be made from contributions first, then conversions, then earnings. In this case the distinction in distribution order is irrelevant since he withdrew the entire account balance. However, his contribution will be tax free, leaving only the $25,000 in earnings as taxable income. The 10% penalty does not apply to this distribution since he qualifies for the higher education exception to the penalty.

Cody is considering establishing a 401(k) for his company. He runs a successful video recording and editing company that employs both younger and older employees. He was told that he should set up a safe harbor type plan, but has read on the Internet that there is the safe harbor 401(k) plan and a 401(k) plan with a qualified automatic contribution arrangement. Which of the following statements accurately describes the similarities or differences between these types of plans? The safe harbor 401(k) plan has more liberal (better for employees) vesting for employer matching contributions as compared to 401(k) plans with a qualified automatic contribution arrangement. Both plans provide the same match percentage and the same non-elective contribution percentage. Employees are required to participate in a 401(k) plan with a qualified automatic contribution arrangement. Both types of plans eliminate the need for qualified matching contributions, but may require corrective distributions.

Safe harbor plans require 100% vesting, while 401(k) plans with QACAs require two year 100% vesting. The matching contributions are different for the plans. Employees are not required to participate in either plan. Both plans eliminate the need for ADP testing, which means that they eliminate the need for qualified matching contributions and corrective distributions.

Bertha, who is 54 years old, spent most of her career in the corporate world and now provides consulting services and serves as a director for several public companies. Her total self-employment income is $500,000. She is not a participant in any other retirement plan today. She would like to shelter as much of her self-employment earnings as possible by contributing it to a retirement plan. Which plan would you recommend? Establish a 401(k) plan. Establish a target benefit plan. Establish a Deferred Comp program for himself. Establish a SEP.

She will be able to defer $56,000 plus the catch up of $6,000 to the 401(k) plan, where she can only contribute $56,000 to the target benefit plan and the SEP. She cannot set up a deferred compensation plan and defer tax.

Which of the following describe benefits usually available under an employer-provided short-term disability plan? Short-term disability coverage will start on the first day when disability is related to an illness. The definition of disability under short-term disability coverage is defined as the inability to perform the normal duties of one's position. Benefits under a short-term disability plan usually extend for one year. Generally, short-term disability coverage will start after sick pay benefits have been provided to a covered employee.

Short-term disability benefits usually start the eighth day of an illness (first day for an accident) and generally last no more than six months.

Marilyn Hayward is the sole proprietor and only employee of unincorporated Graphics for Green Promotions. In 2019, Marilyn established a profit sharing Keogh plan with a 25% contribution formula. As of December 31, 2019, Marilyn has $140,000 of Schedule C net earnings. The deduction for one-half of the self-employment tax is, therefore, $9,891. What is the maximum allowable Keogh contribution that Marilyn can make?

Since she is self employed use 20% of income after half of the self employment tax. Half of the self-employment tax is given, you can skip the 1st step in the self-employment contribution formula (multiplying by 92.35%) 140,000 - $9,891 = 130,109 x 20%*= 26,021.80 *The 20% is from the contribution rate formula: contribution rate / (1 + contribution rate) = self-employment contribution rate .25/ (1+.25) = .20. Note that if the employer has employees and contributes 15% for them, that is the contribution rate to use in this formula.

Which of the following are basic provisions of an IRC Section 401(k) plan? Employee elective deferrals are exempt from income tax withholding and FICA / FUTA taxes. Employer's deduction for a cash or deferred contribution to a Section 401(k) plan cannot exceed 25% of covered payroll reduced by employees' elective deferrals. A 401(k) plan cannot require, as a condition of participation, that an employee complete a period of service greater than one year. Employee elective deferrals may be made from salary or bonuses. I and III only. I and IV only. II and IV only. II, III and IV only.

Statement "I" is incorrect because all CODA plans, including 401(k) plans, subject the income to Social Security and Medicare tax even though Federal and state income tax is deferred by placing the income into the plan. Statements "II", "III" and "IV" are accurate.

Which of the following is/are accurate of a Section 125 cafeteria plan? 30% of the total benefits can accrue to key employees. There must be at least one cash benefit. Deferral of income is not allowed except through a 403(b). Salary reductions can be changed at any time during the year. I only. II only. II and III only. I and IV only.

Statement "I" is incorrect because only 25% of the total benefits can accrue to key employees. Statement "III" is incorrect because deferrals are allowed only through a 401(k) plan. Statement "IV" is incorrect because mid-year changes in reductions are allowed only for qualified changes in status.

Which of the following correctly describes the tax implications of a self-funded accident or medical plan where the employer reimburses the employee directly? In a discriminatory plan, the employer cannot deduct the reimbursements paid to the employee. In a discriminatory plan, a highly-compensated employee must include the excess benefit in his or her income. In a non-discriminatory plan, the benefits received by employees are generally tax free without limitation. In a non-discriminatory plan, the employer can deduct reimbursements to the employee if they are paid to the employee or the employee's beneficiary and are considered reasonable compensation. In a discriminatory plan, benefits received by non-highly compensated employees are generally tax free without limit.

Statement "I" is incorrect because the employer can always deduct the premiums. Statement "II" - The highly compensated employees may be required to pay taxes on all or part of the reimbursements.

Which of the following accurately describes some attributes of non-qualified retirement plans? The employee will pay Table 1 costs each year on an "employer pay all" split dollar life insurance arrangement. The employer can deduct the premiums paid for a split-dollar life insurance arrangement in the year the premiums are paid. Death benefits from a split-dollar arrangement, both the employer and the employee's beneficiary's share, are generally tax free. If the employee's portion of the life insurance premium is greater than the P.S. 58 cost, the excess premiums "rolls forward" to a future year to accurately reflect the employee's cost basis

Statement "II" is incorrect because the employer is unable to deduct any contributions to a non-qualified plan until the employee actually takes constructive receipt. In the traditional split-dollar arrangement, the employer has an interest in the cash values of the split-dollar policy equal to the amount of premiums paid, and therefore, there is never a deduction for premiums paid. Statement "III" - Because no tax deductions are taken for any premiums paid on the policy, the death benefits are tax-free. Statement "IV" - The employee is required to pay the Table 1 cost each year, without regard to premiums paid in previous years.

Joyce is a party to a Qualified Domestic Relations Order (QDRO) relating to her previous spouse's retirement plan. Which of the following statement regarding restrictions related to a QDRO is/are true? Cannot assign a benefit that the plan does not provide. Cannot assign a benefit that is already assigned under a previous order. If the participant has no right to an immediate cash payment from the plan, a QDRO cannot require such a payment. QDRO funds may not be rolled over into a rollover IRA. I and III only. II and IV only. II, III and IV only. I, II and III only.

Statement "IV" is not accurate in that a distribution as a result of a QDRO may be rolled over into an IRA as long as the rollover is accomplished within 60 days of the distribution.

Which of the following transactions by a qualified plan's trust are subject to Unrelated Business Taxable Income (UBTI)? A trust obtains a low interest loan from an insurance policy it owns and reinvests the proceeds in a CD paying a higher rate of interest. A trust buys an apartment complex and receives rent from the tenants. The trust buys vending machines and locates them on the employer's premises. The trust rents raw land it owns to an oil & gas developer. I and II only. I and III only. II and IV only. I, II and IV only.

Statements "I" and "III" are subject to UBTI because income from any type of leverage or borrowing within a plan is subject to UBTI. Additionally, any business enterprise run by a qualified plan is subject to UBTI. Statement "II" is not subject to UBTI (assuming it is not subject to leverage) due to a statutory exemption for rental income. Statement "IV" - The rental of raw land is also exempt. If the plan actually participated in the development of the oil & gas reserves, there would be UBTI.

Carolyn Smart wanted to volunteer full-time and decided to retire from Lotsa Cash Corporation at the age 57, after 15 years of service. She requested a total distribution of her account in the Lotsa Cash Corporation's profit sharing plan and received a check, made payable to her. Her account balance was $60,000 on her final day of employment. Which of the following statements describe the consequences of this distribution? Eligible for 10 year forward averaging Subject to 10% penalty Eligible for Rollover Subject to mandatory 20% withholding Exempt from the 10% early withdrawal penalty

Statements II and V cannot co-exist. She is not old enough for statement I. She is not subject to 10% penalty because this is a qualified plan and she is over 55. Distributions from qualified plans are subject to mandatory 20% withholding.

Myron has a life insurance policy in his qualified plan at work. He has come to you for advice about retirement and other financial planning needs. Which of the following is not correct about the life insurance in a qualified plan? He will be subject to income only if the policy in his qualified plan is a cash value type policy. The policy will be included in his gross estate if he were to die while still working. Part of the proceeds could be taxable to his beneficiary if it is a cash value policy. When he distributes the policy from his plan at retirement, he can convert it to an annuity within 60 days to avoid taxation.

Statements b, c, and d are correct. Statement a is false because all life insurance in qualified plans is subject to income when purchased, regardless of the type.

Sherman, age 52, works as an employee for Cupcakes Etc, a local bakery. Cupcakes sponsors a 401(k) plan. Sherman earns $50,000 and makes a 10% deferral into his 401(k) plan. His employer matches the first 3% deferral at 100% and they also made a 5% profit sharing contribution to his plan. Sherman also owns his own landscaping business and has adopted a solo 401(k) plan. His landscaping business earned $40,000 for the current year. What is the most that Sherman can contribute in the solo plan, assuming his self-employment taxes are $6,000? $19,000 $20,000 $25,000 $27,400

The amount he contributed into his employer's 401k reduces the aggregate amount he can put into a 401k (19k + 6K catch up) An individual can defer up to $19,000 (2019) plus an additional $6,000 catch up for all of their 401(k) and 403(b) plans combined. Since he is 50 or older he can contribute the 19,000 + 6,000 = $25,000. Since he already contributed $5,000 into his employer plan he can still defer $20,000 ($25,000 - $5,000) into the solo plan. The employer contributions in this question are in addition to the employee deferral limit. Employer contribution into the solo plan: self-employment income $40,000 less 1/2 SE tax $3,000 Net $37,000 X 20% employer contribution $7,400 Total contribution to the solo plan = $20,000 + $7,400

Ballistic Laser Operated Weapons company (BLOW) is a defense contractor who develops innovative weapons involving laser-guided systems. The company has maintained a defined benefit plan and a money purchase pension plan for many years. The current benefit formula for the defined benefit plan equals 3% times years of service times the average of the last three years of salary, limited to a maximum benefit of 70%. The money purchase pension plan calls for a 6% contribution for all employees who are covered under the plan. BLOW has been experiencing financial difficulties due to changes in the industry and from competitors and alternative technologies. Based on these challenges, the company is considering changing the benefits under the plans. Which of the following changes would not be permitted under the anti-cutback rules? Changing the benefit accrual for the defined benefit plan from 3% per year to 2% per year for future years. Reducing the money purchase pension plan contribution from 6% to 3% for future years. Decreasing the maximum benefit under the defined benefit plan from 70% to 50% for all future retirees. Switching the vesting for the money purchase pension plan from 3 year cliff to 2 to 6 graduated vestsing.

The anti-cutback rules state that you cannot "cutback" benefits that have been accrued to date. Choice a and b affect future benefits. Choice c will more than likely impact current employees who may have accrued 70% benefits, but who have not yet retired. The change would result in a reduction in benefits and is not permitted. Choice d does is a permitted change and would not result in a reduciton in current vesting.

Which of the following apply to legal requirements for a qualified thrift/savings plan? Participants must be allowed to direct the investments of their account balances. Employer contributions are deductible when contributed. In-service withdrawals are subject to financial need restrictions. After-tax employee contributions cannot exceed the lesser of 100% of compensation or $56,000.

The book doesn't really distinguish difference between 457 and TSP. Notice this says a Q TSP. This is a question about what is right, not about which one is wrong. TSP is after tax contribution This correctly describes the Section 415(c) limits on maximum contributions permitted by law. Participants do not have to be given the right to direct their investments. Employees make after-tax contributions to a thrift; employers don't make contributions. Answer "C" is incorrect because only 401(k) plans have statutory hardship withdrawal requirements, not thrift/savings plans.

Matt is a participant in a profit sharing plan which is integrated with Social Security. The base benefit percentage is 6%. Which of the following statements is/are true? The maximum permitted disparity is 100% of the base benefit level or 5.7%, whichever is lower. The excess benefit percentage can range between 0% and 11.7%. Elective deferrals may be increased in excess of the base income amount. The plan is considered discriminatory because it gives greater contributions to the HCEs. I and II only. I, II and IV only. II only. I, II, III and IV.

The correct answer is A. His base rate is 6% and the social security maximum disparity is 5.7% for 11.7% as the top of his range. Statement "III" is incorrect because integration does not affect voluntary deferrals by employees. Statement "IV" is incorrect because, done properly, integration is NOT considered discriminatory.

Meredith is an executive at Papers Unlimited. As part of her compensation she has a restricted stock plan that allows her to receive 1,000 shares of stock after she completes of 5 years of service. At the time of grant the stock was trading at $2 per share. She made a proper 83b election. She met the vesting requirement 6 months ago when the stock was trading at $35. She has decided to sell her stock. Which of the following is true? If she sells the stock today for $1 per share she is not entitled to a loss. If she sells the stock today for $15 per share she will recognize $13,000 in long term capital gain. If she sells the stock today for $28 per share then she will recognize $26,000 in short term capital gains. If she sells the stock today for $38 per share then she will recognize $3,000 in short term capital gains.

The goal of the 83b election is to get the growth taxed at LTCG When she made the 83b election she would have recognized W-2 income of $2,000 (1,000 × $2). Her holding period would have started at the date of grant. When she met the vesting period she would not have recognized anything since she made the 83b election. If she sold the stock at $1 then she would have had a loss of $1,000 ($2,000 basis - $1,000 sale price). If she sold the stock for $15 then she would have long term capital gain of $13,000 (1,000 × ($15 - $2)). If she sold the stock for $28 then she would have long term capital gain of $26,000 (1,000 × ($28 - $2)). If she sold the stock for $38 then she would have long term capital gain of $36,000 (1,000 × ($38 - $2)).

Angelo's Bakery has 105 employees. 90 of the employees are nonexcludable and 15 of those are highly compensated (75 are nonhighly compensated). The company's qualified profit sharing plan benefits 8 of the highly compensated employees and 40 of the nonhighly compensated employees. Does the profit sharing plan sponsored by Angelo's Bakery meet the coverage test?

The plan meets the ratio percentage test. The percentage of NHC employees covered by the plan is 53.33% and the percentage of HC employees covered by the plan is 53.33%. The ratio percentage of the NHC employees covered by the plan compared to the ratio percentage of the HC employees covered by the plan is 100% (53.33% / 53.33%) which is greater than the ratio requirement of at least 70%.

Richard is covered under his employer's Defined Benefit Pension Plan. He earns $200,000 per year. The Defined Benefit Plan uses a funding formula of Years of Service × Average of Three Highest Years of Compensation × 3%. He has been with the employer for 25 years. What is the maximum defined benefit that can currently be used to determine contributions? $564,000 $150,000 $225,000 $280,000

Think about what this question is asking. It is asking about figuring out the amount the pension will pay out as a annual benefit. Then they need to fund according to that "benefit amount" So what amount are they targeting for the annual income? IN this case its his years of service, compensation and 3%. I'm not sure if this question is worded too well given the formula uses an average of three years income. The maximum defined benefit is the lesser of $225000 (2019) or his compensation. However, the funding formula will limit his defined benefit to $150,000 (25 × 200,000 × .03).

If an employee receiving incentive stock options does not meet the employment time requirement, but receives options as a nonqualifying and exercises them, what will the consequences be? The employee will be required to recognize income immediately upon receipt of the options. The employee will be required to recognize compensation income in the year the option is exercised. If an employee meets the holding period requirement, it does not matter whether he or she meets the employment requirement and the option is qualified. There are no consequences to this circumstance.

This illustrates the difference between the treatment of 'qualified' versus 'nonqualified' stock options. The tax implications are immediate and the income is recognized as soon as the option is exercised rather than when the stock is subsequently sold.

WestN, Inc. sponsors a 401(k) profit sharing plan with a 50% match. In the current year, the company contributed 20% of each employee's compensation to the profit sharing plan in addition to the match to the 401(k) plan. The company also allocated a forfeiture allocation of $4,000. The ADP of the 401(k) plan for the NHC is 4%. Wade, who is age 45, earns $190,000 and owns 19% of the company stock. If Wade wants to maximize the contributions to the plan, how much will he defer into the 401(k) plan?

This is a question about ADP which translates into how much is the HCE able to contribute due to the limitation. Wade is highly compensated because he is more than a 5% owner, so the maximum that he can defer to satisfy the ADP Test requirements is 6% (4% + 2%). Wade is also limited by the 415(c) limit of $56,000. Since the company contributes $42,000 (20% of $190,000 + $4,000 of forfeiture allocations), he only has $14,000 to split between the deferral and the match. Thus, he contributes $9,333* and the match is $4,667, which when added to the $42,000 totals $56,000. 6% of his salary of $190,000 is $11,400. However, he cannot defer this amount due to the 415(c) limit. *When he contributes they match 50%, so for every dollar he contributes 1.5 × that amount goes into the plan. Take 14,000/1.5 = $9,333

A small business owner in a very specialized field, establishes a SEP for his proprietorship after 2008. He has two employees of 2 1/2 years making $80,000 per year. He uses the statutory maximum exclusions for all employees. He earns $100,000 in modified earned income. What is the maximum allowable plan contribution to the owner's account?

This is a self employed person - so remember probably not 25%, need to take into account taxes so use 20% Maximum contribution into a SEP is 25% or $20,000 per employee. The modification for the owner, a self-employed person is, would be: EE contribution % / 1 + EE contribution %. In this case the calculation would be .25 / 1.25 = .2; $100,000 × .2 = $20,000 maximum contribution for the business owner.

One of your clients wants to know the maximum amount that might be allocated to her 401(k) account in the current year. She expects to earn $80,000. What amount can you tell your client will be the maximum total annual additions which could be made to her account? $19,000, the employee elective deferral $20,000, 25% of income $25,000 $56,000

This is a terrible question. Again this is about the maximum deferral, not necessarily specific to her or company. Its assumed that both sources. A company can contribute more than 25% of income, just isn't deductible for them. The section 415 limit is applied to all annual additions. The lesser of 100% of income or $56,000 (2019) is the restriction. This amount includes all company contributions and salary deferral maximums.

Satish has AGI of $66,000 (which is all comprised of earned income). She is single and age 51. Her employer made a $6,300 contribution to her SEP for the current year. What is her available deduction allowed for a Traditional IRA contribution? $26,000 $5,600 $6,000 $7,000

This is a two part question 1)determine if she's an active participant in her company's plan - yes 2)determine if her contribution is deductible based on her income She can contribute to a Traditional IRA since she has earned income and is below 70.5 years old. She is considered an active participant because her employer made a contribution to the SEP on her behalf. Her deduction will be limited because she is within the AGI limitation for a single active participant ($64,000 - $74,000) (2019). She is also entitled to the catch up contribution of $1,000 because she is 50 or older. Therefore, her deductible contribution is: 7,000 × ((66,000 - 64,000)/10,000) = $1,400 is not allowed so $7,000 - $1,400 = $5,600 is permitted. Although the question didn't ask - alternatively, she could contribute to a Roth IRA because she is below the AGI limitation of $122,000 - $137,000 (2019).

In order to be qualified, money purchase plans must contain which of the following? A definite and non-discretionary employer contribution formula. Forfeitures can be reallocated to the remaining participants' accounts in a non-discriminatory manner or used to reduce employer contributions. An individual account must be maintained for each employee of employer contributions. The normal retirement age must be specified.

This is about determining 1) is it a Pension Plan or DC PSP? 2)Is it a DB Pension Plan or DC Pension Plan A normal retirement age must be stated in a defined benefit plan, so Statement "IV" is incorrect. Defined contributions plans (such as a money purchase plan) have retirement benefits which are determined by the value of the individual account whenever the participant retires. Forfeitures may be allocated to employees' individual accounts or used to reduce employer required contributions.

Match the following statement with the type of retirement plan that it most completely describes: " A defined benefit plan that has the appearance of a defined contribution plan" is a... Profit sharing plan. Money purchase plan. SIMPLE IRA. Cash balance plan.

This is asking about a defined benefit plan that looks most like a DC plan. Not asking for DC Pension plan - a bit confusing Cash balance plan. Answers "A" and "C" are incorrect since they are not defined benefit plans. Answer "B" - Money purchase plan is a "pension plan" but it does not provide employees with a defined benefit, only a defined contribution. Answer "D" - Cash balance plan provides a defined benefit (returns are guaranteed by the employer) and the employee receives an "account" to see how much they have.

Jacinth is an executive at Papers Unlimited. As part of her compensation she has a restricted stock plan that allows her to receive 2,000 shares of stock after she completes of 5 years of service. At the time of grant the stock was trading at $4 per share. She did not make the 83b election. She met the vesting requirement 8 months ago when the stock was trading at $28. She has decided to sell her stock. All of the following are true, except: If she sells the stock today for $3 per share she would have an ordinary loss of $50,000. If she sells the stock today for $15 per share she will recognize a capital loss of $26,000. If she sells the stock today for $28 per share then she will not recognize any gain or loss. If she sells the stock today for $38 per share then she will recognize $20,000 in short term capital gains.

This is looking for what is not true. Its a capital loss, not an ordinary loss. so first option is incorrect. The numbers are correct, but its categorized incorrecty. When she met the vesting period she would have recognized W-2 income of $56,000 (2,000 × $28). If she sold the stock at $3 then she would recognize a capital loss of $50,000 ($56,000 basis - $6,000 sale price). If she sold the stock at $15 then she would recognize a capital loss of $26,000 ($56,000 basis - $30,000 sale price). If she sold the stock at $28 then she would not recognize any gain or loss ($56,000 sale price - $56,000 basis). If she sold the stock at $38 then she would recognize a short term gain of $20,000 ($76,000 sale price - $56,000 basis).

Meg has AGI of $1,000,000 (which is all comprised of earned income). She is single and age 50. Her employer offers a 401(k) plan and, although she is eligible to defer, she does not make any deferrals into the plan. The employer made a Qualified Matching Contribution during the current year in order to meet the ADP test. Which of the following statements is true? She can contribute $6,000 to Traditional IRA and deduct all $6,000. She can contribute $7,000 to Traditional IRA and deduct all $7,000. She can contribute $6,000 to Traditional IRA and deduct $0. She can contribute $7,000 to Traditional IRA and deduct $0.

This one is tricky. Even though it said they made a qualified matching contribution, it may not have been specifically to her. A qualified matching contribution would only be made to those employees that actually deferred into the 401(k) plan. Therefore, she would not have received a contribution since she did not defer. Therefore, she is not an active participant. If the problem had said that she received a qualified nonelective contribution then the contributions would have been made to all employees and therefore, she would have received a contribution and would have been an active participant. She can contribute and deduct her contribution to a Traditional IRA since she is not an active participant and therefore not subject to an AGI limitation. She is unable to contribute to a Roth IRA because she is above the AGI limitation of $122,000 - $137,000 (2019). Because she is 50 or older she is allowed to make the $1,000 (2019) catch up contribution.

IRC Section 415(c) applies an "annual addition" limited to certain qualified plans. Which of the following statements is correct? The limit is the lessor of 25% of compensation or $56,000 for the current year. The limit only applies to defined contribution plans. Includable additions include forfeiture reallocations, employer and employee contributions and investment earnings. Salary deferrals are included as part of the annual additions limit.

becareful to not get overwhelmed by the choices. read through the questions. Notice one of the options includes investment earnings - not part of IRC 415 calculation Statement "I" is incorrect as the limit is the lesser of 100% of compensation or the annual limit. Statement "III" is incorrect because investment earnings are never included in the Section 415(c) limit calculation. Statement "IV" is correct as salary deferral contributions by employees is counted against the IRC 415(c) limit.

This is a Single Choice Question; skip ahead to question content Incorrect Correct answerYour answer A Bx C D Confidence Level: LowMediumHigh Continue Difficulty Level: Expert Which of the following statements concerning the use of life insurance as an incidental benefit provided by a qualified retirement plan is (are) correct? The premiums paid for the life insurance policy within the qualified plan will trigger a taxable event for the participant at the time of payment. Under the 25 percent test, if term insurance or universal life is involved, the aggregate premiums paid for the policy cannot exceed 25 percent of the employer's aggregate contributions to the participant's account. If a whole life policy other than universal life is used, however, the aggregate premiums paid for the whole life policy cannot exceed 50 percent of the employer's aggregate contributions to the participant's account. In either case, the entire value of the life insurance contract must be converted into cash or periodic income at or before retirement.

both are true. Every year the plan participant pays income tax on the dollar value of the actual insurance protection -- approximately equal to the term insurance cost. This is commonly called the PS58 cost. The sum of all those costs is the participant's basis.

Which of the following statements concerning COBRA is correct? An employer's plan is exempt from COBRA provisions if the employer averages 25 or fewer employees. Continuation coverage must be available to terminating employees, but not to full-time employees shifting to part-time status. After 36 months, the maximum period for continuation of coverage terminates. The government imposes a non-compliance fine on the employer equal to $10 per day per participant.

look for which one is wrong. COBRA is 18 months. Employers must provide COBRA if they have 20 or more employees. A change in benefit status will trigger COBRA eligibility. COBRA non-compliance carries a penalty of $100 per day per participant.

Your clients, Nick and Betty Jo Byoloski, have come to you with some questions. She has been an employee of April Corporation for several years and received some stock options as compensation at times. He has worked with April Corporation as a consultant on several jobs over the last few years and was paid in part with stock options. Nick and Betty Jo want to know more about their situation regarding the options. What can you tell them? Betty Jo's options are qualified and Nick's options are non-qualified. Nick's options are non-qualified and Betty Jo's options are non-qualified. Nick's options are non-qualified and Betty Jo's are either qualified or non-qualified. Betty Jo's options are non-qualified and Nick's options are qualified.

non employees can only have NQL ISO, whereas employees we do not know for sure can be Q or NQL.

Which of the following is true regarding qualified incentive stock options? No taxable income will be recognized by the employee when the qualified option is granted or exercised. The income from sale of the qualified option will always be taxed as capital gains when the stock is sold. The income from sale of the qualified option will be taxed as ordinary income regardless of when the stock is sold. The employer will not be able to deduct the bargain element of the option as an expense under any circumstance. For favorable tax treatment the option must be held two years and the stock for one year after exercise.

only I & V are correct In Statement "II," be careful of "always"! In Statement "III," if held longer than one year, they receive capital gains treatment. In Statement "IV," under most circumstances, the bargain element is deductible. There are exceptions when certain qualifications have not been met for deductibility, such as time employed, time to exercise in excess of rules, etc.

Your client has been a business owner for six years. He recently established a SEP for his business. He has two employees of 24 months making $22,000 per year. He asked you to use the statutory maximum exclusions for all employees, including himself. He has self-employment earnings of $78,000. Assume he has self-employment tax of $11,000. The maximum plan contribution to the owner's account will be:

remember to remove the self employment tax then use 20% Maximum contribution is 25%. Because he is self-employed, the "S/E haircut" is required. The haircut is calculated at $78,000 less 1/2 of 11,000 ($5,500) = 72,500; then multiply by 20% (.25/1.25). Therefore, the maximum contribution is $14,500.


Kaugnay na mga set ng pag-aaral

art appreciation test 4- artists match the painting title

View Set

ІТХВ ПИТАННЯ ДО ІСПИТУ

View Set

Sociology Chapter 12, 13, and 16

View Set

Sociology Lesson 6- Sex and Gender

View Set

Biology 104 Homework 7 Mastering Biology

View Set